Solving Cartesian Equation: r= 9 cosθ

  • Thread starter Thread starter knv
  • Start date Start date
  • Tags Tags
    Cartesian
Click For Summary
To convert the polar equation r = 9 cosθ into Cartesian form, the correct equation is x² + y² = 9x. The user initially derived (x - 9/2)² + y² = (9/2)² but faced issues with online homework submission. The problem was identified as a formatting issue rather than a mathematical error. The discussion highlights the importance of presenting answers in the required format for online submissions. The final correct form simplifies the equation effectively.
knv
Messages
17
Reaction score
0
1. Find a Cartesian equation to represent the curve r = 9 cosθ



2. I know that rcosθ= x and cos θ= x/r



3. I got (x-9/2)^2 +y^2 = (9/2)^2 but its coming up wrong when I put it into our online homework. Can anyone help me?
 
Physics news on Phys.org
hi knv! :smile:

(try using the X2 button just above the Reply box :wink:)
knv said:
1. Find a Cartesian equation to represent the curve r = 9 cosθ
I got (x-9/2)2 +y2 = (9/2)^2

how?? :confused:

anyway, just multiply by r …

r2 = 9rcosθ :smile:
 
knv said:
1. Find a Cartesian equation to represent the curve r = 9 cosθ



2. I know that rcosθ= x and cos θ= x/r



3. I got (x-9/2)^2 +y^2 = (9/2)^2 but its coming up wrong when I put it into our online homework. Can anyone help me?

Your result looks correct to me. It probably has something to do with how you "simplify" it before you type in the answer. Maybe something simple like writing the right side as 81/4?
 
the answer was more simple than I thought.

x2+y2=9x


Thanks!
 
knv said:
the answer was more simple than I thought.

x2+y2=9x


Thanks!

So your original answer was correct, just not in the form the software wanted.
 
yes. I always get things wrong just because its not in the form the software wanted. Miss when we could turn homework in on paper haha
 
Question: A clock's minute hand has length 4 and its hour hand has length 3. What is the distance between the tips at the moment when it is increasing most rapidly?(Putnam Exam Question) Answer: Making assumption that both the hands moves at constant angular velocities, the answer is ## \sqrt{7} .## But don't you think this assumption is somewhat doubtful and wrong?

Similar threads

  • · Replies 1 ·
Replies
1
Views
2K
  • · Replies 1 ·
Replies
1
Views
2K
  • · Replies 3 ·
Replies
3
Views
2K
  • · Replies 3 ·
Replies
3
Views
2K
Replies
3
Views
1K
  • · Replies 3 ·
Replies
3
Views
2K
  • · Replies 2 ·
Replies
2
Views
2K
Replies
1
Views
1K
  • · Replies 2 ·
Replies
2
Views
3K
  • · Replies 4 ·
Replies
4
Views
2K